LSAT and Law School Admissions Forum

Get expert LSAT preparation and law school admissions advice from PowerScore Test Preparation.

 Administrator
PowerScore Staff
  • PowerScore Staff
  • Posts: 8929
  • Joined: Feb 02, 2011
|
#104146
Complete Question Explanation

Flaw in the Reasoning. The correct answer choice is (C).

Answer choice (A):

Answer choice (B):

Answer choice (C): This is the correct answer choice.

Answer choice (D):

Answer choice (E):

This explanation is still in progress. Please post any questions below!
 ikim10
  • Posts: 30
  • Joined: Nov 28, 2022
|
#105099
I was stuck between (A) and (C) on this question, and ended up choosing (A). I thought it was (A) because I thought the argument was confusing the presence of regulations and protecting endangered species.

I diagrammed it something like:

Premise: regulations present :arrow: endangered species protected
Premise: regulations present :arrow: landowners discouraged
Conclusion: regulations NOT present :arrow: endangered species protected

I thought (C) was wrong because I thought "certain factors tend to produce an effect" was referring to regulations (certain factors) tend to discourage landowners from protecting endangered species) (effect). However, the second part of the sentence confused me. I guessed that "countervailing effects" meant an opposite effect that canceled out the other one, but couldn't reconcile it with the substitution I made for the first part of (C).

Could you please explain why (A) is wrong and how (C) is correct, with the proper references for each bit? Thank you!
 Luke Haqq
PowerScore Staff
  • PowerScore Staff
  • Posts: 795
  • Joined: Apr 26, 2012
|
#105196
Hi ikim10!

For (C), I've bracketed what the answer choice seems to be referring to: "It unjustifiably overlooks the possibility that even if certain factors [lifting the regulations] tend to produce a given effect [it takes away an incentive for people not to protect endangered species], they may be likely to produce stronger countervailing effects as well [perhaps lifting the regulations will lead to the rapid extermination of the endangered species]." In other words, the land developer sees that the proposed course of action might have one beneficial effect but overlooks the possibility of countervailing effects (i.e., offsetting effects) of this course of action.

For answer choice (A), I'm not entirely certain what each component of that answer choice is referring to in the stimulus. There's nothing in the stimulus, for example, that is saying that the regulations are required or necessary in order for endangered species to continue to exist.

Get the most out of your LSAT Prep Plus subscription.

Analyze and track your performance with our Testing and Analytics Package.